MIXED SUBJECT QUESTIONS - SET 1

Ace your homework & exams now with Quizwiz!

Question 1 A city has many parks, as well as a country club with a golf course. While the city's parks are accessible to all without a fee, the city charges a $1,000 application fee and $100 per month dues to belong to the country club. A resident of the city wishes to join the country club but cannot afford the application fee or monthly dues. If the resident brings suit against the city on the ground that the fee and dues discriminate against the poor in violation of the Equal Protection Clause, who likely will prevail? (A) The resident, because a person cannot be deprived of a public right or benefit on the basis of inability to pay. (B) The resident, because the poor qualify as a protected class. (C) The city, because only de jure discrimination against the poor has been held to violate the Equal Protection Clause. (D) The city, because the membership privilege is not an important enough deprivation.

1. Constitutional Law Answer to Question 1 (D) The city will prevail because the membership privilege is not a significant enough deprivation to implicate the Equal Protection Clause. Only the denial of particularly important rights (such as a marriage license) to those unable to pay for them has been held to violate equal protection. Therefore, (A) is wrong. (C) is wrong because a number of de facto discriminations against the poor have been held to violate equal protection. (B) is an incorrect statement of the law. The Supreme Court has never held that wealth alone is a suspect classification. Only when the lack of wealth prevents a person from exercising a fundamental constitutional right will equal protection issues be raised.

Question 10 The police entered a neighborhood suffering from increased illegal drug activity, accompanied by a dog trained to sniff out cocaine. The police entered the backyard of a home and brought the dog to an area immediately outside the back door. The dog acted as if it smelled cocaine. The officers knocked on the back door and a man answered the door and let them in. He was immediately placed under arrest. After a brief search, the police officers found and confiscated a small quantity of cocaine from the bedroom closet. The man is charged with possession of cocaine. At trial, he moves to prevent introduction of the cocaine into evidence. This motion will most probably be: (A) Granted, because, under the circumstances, the police activity violated the man's reasonable expectation of privacy. (B) Granted, because this kind of detection by a trained dog has not been scientifically verified and cannot be the basis for probable cause. (C) Denied, because the man allowed the police officers to enter his home. (D) Denied, because the search was incident to a valid arrest.

10. Criminal Law Answer to Question 10 (A) The man's motion will be granted because he had a reasonable expectation of privacy in his bedroom closet. The search of the closet was not based on a valid warrant or circumstances justifying an exception to the warrant requirement. (B) is wrong. There is no such rule of law. (C) is wrong; allowing the police to enter the house is no "consent" to search the bedroom closet. (D) is wrong; a search of the closet would be outside the area of the immediate control of the defendant and could not be justified as a search incident to an arrest.

Question 11 A state conducted a study in which it found that many parents were moving out of certain areas of the state because they could not find affordable day care. As a result, the state legislature enacted a statute that created an agency to operate subsidized day care centers throughout these areas. These day care centers would charge parents a nominal fee compared to private day care centers in the area. A private day care center has discovered that its business has fallen off about 30% since the agency started operating a day care center in its city. It brings an action seeking to enjoin the operation of the agency. What will be the probable outcome of this litigation? (A) The center will win, because operation of a state agency in competition with a private business violates the Due Process Clause of the Fourteenth Amendment. (B) The center will win, because permitting some citizens to do business with a state business while others do business with a private business violates the Equal Protection Clause of the Fourteenth Amendment. (C) The center will lose, because it lacks standing to challenge the legislation, having suffered no direct injury. (D) The center will lose, because the agency is a valid exercise of the state's powers.

11. Constitutional Law Answer to Question 11 (D) Providing day care centers for its citizens is clearly within the reserved police powers of the state. Because this is facially neutral economic and social legislation, the state simply needs a rational basis for its actions, and that is present. (A) and (B) are, accordingly, wrong. A state may undertake traditionally private business activities, and the analysis for both due process and equal protection purposes is rational basis review, under which the state will prevail. (C) is not a good answer because the center has suffered a direct injury (lost business) caused by the state government action. It has standing to challenge the legislation, but standing has nothing to do with the merits of the claim, which it will lose.

Question 12 The defendant was on trial for trying to sell a stolen antique ring to an antique dealer. The defendant claims that she had bought the ring from a street vendor earlier on the day of her arrest. The prosecution calls to the stand a woman who had given a party in her home which the defendant attended, and who had discovered later that evening that her antique ring was missing from her bedroom. She seeks to testify that her friend, who is now backpacking in another country, told her three days after the party that she had seen the defendant the previous evening in a restaurant wearing a ring that looked exactly like the woman's ring. The defense attorney objects. The objection should be: (A) Overruled, because the statement is a present sense impression. (B) Overruled, because the statement is relevant and the friend is unavailable, having gone backpacking in another country. (C) Sustained, because this is circumstantial evidence within circumstantial evidence. (D) Sustained, because the statement is hearsay not within any exception.

12. Evidence Answer to Question 12 (D) The friend's comment is an out-of-court statement offered for its truth, and it does not fall within a recognized exception to the hearsay rule. (A) is incorrect because a present sense impression must be communicated while perceiving the event or object, or immediately thereafter. The friend was telling the woman about something she had seen the previous evening. (B) is incorrect because, even if the statement is relevant, it is still hearsay, and the friend's unavailability does not bring it within an exception to the hearsay rule. (C) does not state a reason for excluding the evidence; circumstantial evidence is admissible.

Question 13 A homeowner was trimming his sidewalk-bordering hedge when a wasp began attacking him. The homeowner attempted to hit the wasp. During one of his swats, the homeowner struck a jogger in the face. The jogger, reacting to the unexpected blow to his head, reached into his pocket and pulled out a knife. The homeowner tried to shield himself by raising his arms in front of his body, but the jogger was able to stab the homeowner, seriously injuring him. If the jogger is prosecuted for aggravated battery, he probably will be found: (A) Not guilty, because he believed the homeowner was attacking him. (B) Not guilty, because he was adequately provoked by the homeowner. (C) Guilty, because he used a deadly weapon. (D) Guilty, because he intentionally created a reasonable apprehension of imminent bodily harm in the homeowner.

13. Criminal Law Answer to Question 13 (C) The jogger's act constituted an unlawful application of force to the person of another and is, thus, a battery. Use of a deadly weapon in the commission of a battery elevates the crime to aggravated battery. (A) is wrong because a person must reasonably believe that he is faced with imminent death or great bodily harm in order to use deadly force. The accidental blow struck by the homeowner would not rise to that level. (B) is incorrect. Although adequate provocation (e.g., being subjected to a serious battery or a threat of deadly force) is enough to reduce a killing to voluntary manslaughter, there is no rule justifying a battery based on adequate provocation. The fact that the jogger was unexpectedly struck by the homeowner does not justify his battery of the homeowner. (D) is wrong because it describes a type of assault, not battery. If there has been an actual touching of the victim, a battery has been committed. In this case, the jogger actually touched the homeowner when he stabbed the homeowner with the knife. Therefore, the jogger will be guilty of battery.

Question 14 A rancher owned a 100-acre tract of land. Eighty acres had been devised to him by his father, and the rancher had acquired title to an adjacent 20 acres from a prospector by adverse possession. The rancher entered into a land sale contract in which he promised to convey the 100 acres to an investor. The contract listed the description of the property and was otherwise definite and certain as to the transaction, but the contract did not state the nature of the title that the rancher was to convey to the investor. At the time of closing, the investor paid the purchase price and accepted the deed conveyed by the rancher. Six months later, the prospector returned to the area and brought a successful action in ejectment against the investor for the 20 acres. The investor now sues the rancher for damages. Which of the following statements would most accurately describe the investor's rights? (A) The investor's rights are based on the implied covenant contained in a marketable title. (B) The investor could bring an action for reformation of the deed with an abatement of the price of the land. (C) The terms of the investor's deed control the rancher's liability. (D) The investor could bring an action against the rancher for fraud.

14. Real Property Answer to Question 14 (C) The terms of the deed control the rancher's liability. While the contract did not specify the quality of title, the law implies that a marketable title is to be conveyed, and since a title based on adverse possession is not marketable, although it may be good, the investor could have refused to perform. Once the investor accepted a deed, however, the doctrine of merger comes into play and the contract is merged into the deed. Any contract provisions for quality of title, express or implied, are no longer effective. Thus, (A) is incorrect. The investor must look to the terms of the deed for his rights. (B) is incorrect because the court will not rewrite the deed. The facts do not indicate that the rancher made any misrepresentations, and so (D) is incorrect.

Question 15 A tourist from a foreign country rented a car from a car rental company that is incorporated and has its principal place of business in State A. The tourist then was in a traffic accident with another driver, a citizen of State B. The State B driver filed an action in federal district court against both the tourist and the rental company, alleging that both were negligent and liable for the injuries the State B driver sustained in the accident. The rental company believes that both the tourist and the State B driver were negligent and wants to recover from both for the extensive damage done to its car. Are the rental company's claims against the State B driver and the foreign tourist compulsory such that it must assert them in the State B driver's pending federal action? (A) Both claims are compulsory. (B) The claim against the State B driver is compulsory, but the claim against the foreign tourist is not. (C) The claim against the foreign tourist is compulsory, but the claim against the State B driver is not. (D) Neither claim is compulsory.

15. Civil Procedure Answer to Question 15 (B) The rental company's claim against the State B driver is compulsory, but the claim against the tourist is not. Counterclaims are compulsory if they arise from the same transaction or occurrence as the claim asserted against the pleading party. Since the rental company's claim arises from the same transaction or occurrence as the State B driver's claim against it, the counterclaim is compulsory. Therefore (C) and (D) are incorrect. The rental company's claim against the tourist is a cross-claim, and cross-claims are not compulsory. Therefore (A) is incorrect.

Question 16 To help alleviate discrimination in private contracts, Congress passed a bill providing: It shall be unlawful to discriminate against minority race members in the making and enforcement of any public or private contract, of every kind whatsoever. Any person whose rights under this statute are violated may bring a cause of action against the party that has so violated the person's rights in the federal district court for the district in which he resides, seeking treble damages or $1,000, whichever is greater. Several large banks that have been accused of discriminatory loan practices challenge the federal statute. If the court finds that Congress had the power to enact the statute, the court most likely will find that the power arose from: (A) The Contract Clause. (B) The Thirteenth Amendment. (C) The Fourteenth Amendment. (D) The Commerce Clause.

16. Constitutional Law Answer to Question 16 (B) The court most likely will find that Congress had the power to enact the legislation under the Thirteenth Amendment. The Thirteenth Amendment simply provides that neither slavery nor involuntary servitude shall exist within the United States and gives Congress the power to adopt appropriate legislation to enforce the proscription. Since the amendment is not limited to proscribing state action, Congress may adopt legislation regulating private parties. Under the amendment, the Supreme Court has allowed Congress to prohibit any private conduct that Congress deems to be a "badge" or "incident" of slavery, and has upheld statutes regulating private contracts. [See, e.g., Runyon v. McCrary (1967)] (A) is not a good basis for the statute because the Contract Clause is a limitation on states' rights to modify contracts retroactively; it is unrelated to Congress's power to regulate private contracts. (C)—the Fourteenth Amendment—is incorrect. The Fourteenth Amendment prohibits states from discriminating on the basis of race; it does not extend to private conduct. [See United States v. Morrison (2000)] (D)—the Commerce Clause—might also be a basis for the legislation here, but it is not as good an answer as (B) because the commerce power is limited to transactions that either in themselves or in combination with other activities have a substantial economic effect on interstate commerce, and by its terms the legislation here can reach wholly intrastate transactions. The interstate commerce requirement is a limit on congressional legislation and no such limit is present under the Thirteenth Amendment. Therefore, the Thirteenth Amendment is a better basis for the legislation here.

Question 17 The owner of a sporting goods store noticed that her tent stock was running low. After consulting various manufacturers' catalogues, she decided to order from a large manufacturer of camping equipment whose catalogue listed the 9 x 12 tent that she wanted, at a cost of $70. On April 1, the store owner phoned the manufacturer and placed her order for 10 tents with the manufacturer's sales agent. The next day, the manufacturer mailed the store owner a letter informing her that the tents were now $72 and that they would be shipped to her on April 16. The store owner received the letter on April 4, but she never responded. On April 15, she received a catalogue from another tent company showing tents similar to the ones that she ordered, but for a cost of $50. She immediately called the manufacturer with whom she had placed her order to cancel it. Nevertheless, the manufacturer shipped the tents to her on April 16. If the manufacturer sues the store owner to enforce the contract, who will prevail? (A) The store owner, because there was no meeting of the minds regarding the price term. (B) The store owner, because her promise was not in writing. (C) The manufacturer, because its April 2 letter was sufficient to bind the store owner. (D) The manufacturer, because the store owner's phone call on April 15 to cancel is proof that there was a contract.

17. Contracts Answer to Question 17 (C) The manufacturer will prevail. This fact situation is governed by U.C.C. section 2-201 because it involves a written confirmation of an oral agreement. In a contract between merchants under U.C.C. section 2-201, a writing in confirmation of the contract (even if it varies from the original offer) that is sufficient to bind the sender will also satisfy the Statute of Frauds against the recipient unless a written objection is made within 10 days, which was not done by the buyer here. Hence, (B) is wrong. (A) is wrong because an oral contract was created when the store owner placed the order with the manufacturer's agent or the store owner's offer was accepted when the manufacturer sent the confirmation. Under Article 2, an acceptance containing different or additional terms is still effective as an acceptance unless made expressly conditional on assent to those terms. (D) is wrong because the store owner would be bound regardless of whether she made the phone call to cancel.

Question 18 A state enacted legislation providing for the testing of all high school students in the performance of certain adult-world tasks. In public schools, the test was administered at the school by government employees. In private schools—both religious and secular—the test was administered by school personnel, and the schools were reimbursed for the costs of administering the tests and reporting the results to the state. An association with appropriate standing filed suit seeking to enjoin the reimbursement to the religious schools, claiming that the reimbursement violates the Establishment Clause of the First Amendment to the United States Constitution. The trial court should: (A) Grant the injunction, because direct state payment to religious schools violates the Establishment Clause. (B) Grant the injunction, because the state may not require the religious schools to conduct tests that violate their religious principles. (C) Deny the injunction, because the reimbursement has a secular purpose that neither advances nor inhibits religion and that will not entangle the state in religion. (D) Deny the injunction, because the tests do not involve matters of religion.

18. Constitutional Law Answer to Question 18 (C) In Committee for Public Education and Religious Liberty v. Regan (1980), the United States Supreme Court held that a similar statute passed by the state of New York did not violate the Establishment Clause because it had a secular legislative purpose. Its principal effect did not advance or inhibit religion, and it did not foster an excessive government entanglement with religion. Here, our facts show a state law that provides for secular testing and reimbursement for testing administration costs to religious schools. As such, the state law does not violate the Establishment Clause because it had a secular legislative purpose (namely, reimbursing schools for secular testing that was conducted in all schools—religious and secular alike), its principal effect did not advance or inhibit religion, and it did not foster an excessive government entanglement with religion. (A) is therefore incorrect; aid can be given to religious schools that pass this test. (B) is wrong. It misstates the facts given, which make no mention of a specific religious group. Rather, the suit was filed by an association challenging the reimbursement. (D) is irrelevant. The question is not whether the tests involve "matters of religion," but whether the reimbursement program impermissibly advances religion.

Question 19 After reading an article in a hunting magazine detailing a state's expanded season for the hunting of grizzly bears, a hunter called his nephew to see if he wanted to take a trip to the state to hunt grizzly bears. His nephew agreed. Unknown to the hunter and his nephew, the article in the magazine listed an incorrect ending date for the expanded grizzly bear hunting season; the hunting season had expired the day before. While still in their pickup truck driving to a campsite in the state, the hunter and his nephew were pulled over by a state trooper. They volunteered that they were on their way to hunt grizzly bears and were promptly arrested. A state statute made hunting bears out of season a strict liability offense. If the hunter and his nephew are charged with conspiracy to hunt grizzly bears out of season, they will be: (A) Acquitted, because there was no agreement. (B) Acquitted, because they had not intended to commit a crime. (C) Convicted, because hunting grizzly bears out of season is a strict liability offense. (D) Convicted, because they took action in furtherance of the conspiracy.

19. Criminal Law Answer to Question 19 (D) The hunter and his nephew will be convicted because they took action in furtherance of the conspiracy. Conspiracy requires (i) an agreement between two or more persons; (ii) an intent to enter into an agreement; and (iii) an intent to achieve the unlawful objective of the agreement. Here, the hunter and his nephew, with the intent to do so, agreed to hunt grizzly bears on a particular date when hunting was illegal. While they did not know that hunting was illegal on that date, their ignorance of the state law on that point does not negate their intent to commit the act and is not a defense. The majority rule is that the parties to a conspiracy need not have been aware that their plan was an illegal one. While a few courts hold that the conspirators must have know that their objective was criminal unless the crime was malum in se ("corrupt motive" doctrine), most courts reject this exception. Hence, the hunter and his nephew can be convicted of conspiracy. (A) is incorrect because the facts show an agreement between the two. (B) is incorrect because they intended to commit the act that constituted the crime. As discussed above, the fact that they mistakenly believed that their objective was legal is not a defense in most jurisdictions. (C) is incorrect because when a defendant is charged with conspiracy, even conspiracy to commit a strict liability offense, an intent to achieve the objective of the agreement must be shown.

Question 2 To provide low-cost housing to the unemployed, a city has a policy of leasing empty city-owned buildings to social agencies that promise to convert or rehabilitate the buildings into habitable, low-cost apartments and to pay the city 10% of any net profit made from rentals. A church entered into such an agreement with the city and converted one of the city's abandoned office buildings into 50 small, low-cost apartments. The lease agreement used by the church provides, among other things, that the lessee must affirm a belief in God. The lease agreement was submitted to the city for approval prior to its use by the church, and it was approved. On the first day that the church made the apartments available for rent, the plaintiff, an avowed atheist, applied to lease a unit. The plaintiff's application was denied for the sole reason that the plaintiff refused to affirm a belief in God. If the plaintiff brings suit against the church on the ground that the required affirmation of a belief in God violates the plaintiff's constitutional rights, who likely will prevail? (A) The plaintiff, because denial of a lease to atheists has been held to hinder the free exercise of religion. (B) The plaintiff, because the purpose and effect of the church's policy results in a violation of the Establishment Clause. (C) The church, because freedom of religion is not protected against acts of private individuals or groups or a private institution. (D) The church, because as an atheist, the plaintiff has no standing to challenge the lease requirement on religious grounds.

2. Constitutional Law Answer to Question 2 (B) The plaintiff will likely prevail. The First Amendment prohibits laws respecting the establishment of religion. Governmental action that does not contain a sect preference will pass muster under the Establishment Clause if it has a secular purpose, its primary effect neither advances nor inhibits religion, and it does not require excessive government entanglement with religion. The church's action will be considered to be state action here because of the significant involvement between the church and the city. (The city is leasing the building to the church, the church shares profits with the city, and the church submitted its lease forms to the city for approval.) Requiring a lessee to affirm a belief in God appears to have no secular purpose. Moreover, its primary effect probably is to advance religion. Therefore, the church's action will be found to have violated the Establishment Clause. (A) is wrong because there simply is no such Supreme Court holding. (C) is wrong because, as was pointed out above, state action can be found because of the city's significant involvement in the apartment building at issue. (D) is wrong because a person asserting a violation of the Establishment Clause does not have to allege infringement of a particular religious freedom in order to have standing; it is enough that the person is directly affected by the government action challenged.

Question 20 While practicing their target shooting at the firing range, a man and woman got into an argument that almost erupted into physical combat, except that they were restrained and separated by bystanders. Later, in the parking lot of the range, the man shot the woman in the shoulder. Bystanders who rushed to the scene immediately after hearing the man's shot found the woman on the pavement with a black metal flashlight in her hand. The woman's pistol was in her locker at the firing range. At the trial of the woman's civil action for battery against the man, the woman established that the man intentionally shot her. In defense, the man testified that the woman approached him, saying, "We'll settle this once and for all, right now," and raised an object toward the man. He testified that he feared that the woman was about to shoot him with a pistol, so he fired in self-defense. Assuming that the jury decides that the man is telling the truth, to find in his favor the jury also will need to find: (A) No additional facts. (B) That a reasonable person in the same circumstances would have believed that the woman was about to shoot. (C) That the woman was at fault in raising a black object toward the man while threatening him. (D) That the woman was the original aggressor.

20. Torts Answer to Question 20 (B) If the man prevails, it will be because the jury determined that he acted reasonably under the circumstances. One may act in self-defense not only where there is real danger but also where there is a reasonable appearance of danger. An honest but mistaken belief that the woman was about to shoot would justify the use of deadly force by the man if a reasonable person would have acted similarly under those circumstances. The test is an objective one—an honest belief alone is not sufficient. Thus, (A) is incorrect. (C) is incorrect because the woman's fault is not the determining factor—the reasonableness of the man's belief governs for self-defense. (D) is incorrect because it does not resolve whether the man had the right to use deadly force. Even if the man started the altercation at the range, he would have the right to use deadly force if the woman escalated the fight with deadly force.

Question 21 An employee of a grain company brought suit against his employer for injuries that he suffered in a fire and explosion in a grain elevator. The employer filed a counterclaim against the employee for damages, and alleged that the employee was contributorily negligent in that he cut through some electrical wires while working in the elevator, and the sparks from those wires caused the explosion. The employee denies these allegations. His employer calls the grain elevator operator, who was also working in the elevator, and he testifies that he helped the employee and his assistant out of the elevator soon after the explosion. The elevator operator intends to testify that, at that time, the assistant told him that the employee "should have been able to tell that that wire was hot." The injured employee's attorney objects to the elevator operator's testimony concerning this conversation. The trial judge should rule that the testimony is: (A) Inadmissible, because it is hearsay not within any exception. (B) Inadmissible, because it is improper opinion evidence. (C) Admissible, as an admission. (D) Admissible, as being a prior inconsistent statement.

21. Evidence Answer to Question 21 (A) The testimony should be ruled inadmissible because it is hearsay and does not clearly fall within a hearsay exception under the facts given. (B) is wrong because the opinion was based on the perception of the witness and would be helpful to the determination of a fact in issue—the negligence of the injured employee. (C) is wrong because the assistant is not a party. Furthermore, the assistant's statement is not a vicarious admission of the employee because the assistant is employed by the grain company, not the employee. (D) is wrong because the assistant did not testify and therefore the statement cannot qualify as a prior inconsistent statement. The statement is clearly hearsay. It could be argued that it qualifies as an "excited utterance," but there are not enough facts to establish this; also, "admissible as an excited utterance" is not one of the four picks. Therefore (A) is the winner because (B), (C), and (D) are clearly wrong.

Question 22 A restaurant owner in State A bought two large freezers from a manufacturer of commercial refrigeration equipment with its principal place of business in State B. Within one week and after being fully stocked with meat, one of the freezers broke down. The restaurant owner filed a state-based products liability action against the manufacturer in federal court in State A, and included a demand for a jury trial. Under the law in State A, jury verdicts do not need to be unanimous, but the Federal Rules of Civil Procedure require jury verdicts to be unanimous. At trial, the restaurant owner makes a motion asking the court to apply the State A law. How should the court rule on the motion? (A) Grant the motion, because applying the federal rule may change the outcome of the case. (B) Grant the motion, because, when a federal court has diversity jurisdiction, it is required to apply the substantive law of the state in which it is sitting. (C) Deny the motion, because the Federal Rules of Civil Procedure apply in federal court as long as they are consistent with the Rules Enabling Act and not unconstitutional. (D) Deny the motion, because the Supreme Court's balancing factors indicate that federal law should apply.

22. Civil Procedure Answer to Question 22 (C) The court should deny the motion. Under the Erie doctrine, when a state law-based claim is brought in federal court based on diversity of citizenship, the federal court generally applies the substantive law of the state in which it is sitting. However, where a specific federal statute or the Federal Rules of Civil Procedure are on point, the federal court must apply federal procedural law as long as the federal rule is valid. Under the Rules Enabling Act, a Federal Rule is valid if it deals with "practice or procedure" and does not "abridge, enlarge, or modify" a substantive right. Here, there is a specific federal procedural rule that is on point [Fed. R. Civ. P. 48], which requires jury verdicts to be unanimous, unless the parties agree otherwise. Since there is no evidence of agreement, the federal procedural rule will apply, and the motion should be denied. (A) is wrong because it states the wrong conclusion, and it incorrectly cites the "outcome determination" test. This is the test that may be applied when there is no federal procedure law on point, and it is unclear whether the state matter is substantive or procedural. As stated above, there is a federal procedural rule on point. Therefore, this test does not apply. Similarly, (D) is wrong because it is referring to the balance of interests test, which is another test that may be applied when there is no federal procedural law on point, and it is unclear whether the state matter is substantive or procedural. This test is not applicable here, however, because there is a federal procedural rule on point. (B) is wrong because, although it is a true statement of law that a federal court with diversity jurisdiction is required to apply the substantive law of the state in which it is sitting, the requirements for a jury verdict is a procedural rule, not a substantive rule. Therefore, as set forth above, the federal procedural rule governs.

Question 23 A driver was operating her car on a city street when she was stopped by a police officer for speeding. As the police officer reached the driver's car, he saw her put something into her purse. The officer told the driver, "Ma'am, you were speeding; that's why I stopped you. I'd like your driver's license, and, by the way, what did you just put into your purse?" The driver responded, "It's just a marijuana cigarette, but don't worry, I've only had two and my driving judgment hasn't been impaired." The officer took her purse, removed the "joint," and charged the driver with possession of marijuana as well as speeding. At the driver's trial for marijuana possession, the prosecution seeks to introduce the marijuana cigarette into evidence. The driver's attorney moves to suppress the evidence. The defense motion should be: (A) Granted, because the cigarette is fruit of the poisonous tree. (B) Granted, because the police officer did not have a valid search warrant. (C) Denied, because the police officer's asking about the contents of the driver's purse did not constitute custodial interrogation. (D) Denied, provided the police officer had a reasonable suspicion of criminal activity.

23. Criminal Law Answer to Question 23 (C) The defense motion should be denied because the driver was not in custody when she made the statement. Persons temporarily detained for routine traffic stops are not in custody for Miranda purposes. Therefore, the driver was not entitled to Miranda warnings, and her statement about the marijuana was not tainted. Her statement thus properly provided the probable cause for the search of her purse. (A) is therefore wrong. (B) is wrong because this case falls within the automobile exception to the warrant requirement. (D) states the test for a stop, not a search. An automobile search requires probable cause.

Question 24 A man and a woman met in a bar. The woman told the man that she greatly admired the diamond stickpin he had in his lapel. "Oh, this," the man laughed. "It's no diamond; it's only a piece of glass." The woman acknowledged his statement, but kept commenting on how nice it looked. After further conversation, the man orally agreed to sell the stickpin to her for $510. They agreed that in two days, he would bring the stickpin to the same bar, and the woman would bring the $510 in cash. The man duly appeared with the pin, but the woman failed to appear. The man filed suit against the woman for $510. The woman's best defense is: (A) $510 was an unconscionable amount to pay for a piece of glass. (B) The agreement was not supported by consideration. (C) The agreement violated the Statute of Frauds. (D) Neither the woman nor the man was a merchant.

24. Contracts Answer to Question 24 (C) A promise for the sale of goods for $500 or more is not enforceable under the Statute of Frauds unless evidenced by a writing signed by the party to be charged. Uniform Commercial Code section 2-201(1) applies to the agreement between the man and the woman because the stickpin is a tangible, movable item of property, which is the definition of goods under the Code. The Statute of Frauds provision applies regardless of the fact that neither the man nor the woman is a merchant. The woman's promise, therefore, cannot be enforced by the man because it was an oral promise to purchase goods that cost more than $500. (A) is incorrect because the concept of unconscionability allows avoidance of a contract only where the terms are so one-sided as to indicate unfair surprise or a contract of adhesion. Here, neither party had superior bargaining power, and the woman knew exactly what she was buying when she made the agreement. (B) is incorrect because the agreement was a bargained-for exchange in which both parties would suffer detriments by giving something up. Regardless of whether the pin was objectively worth much less than $510, its appearance gave it a higher value in this transaction, and a court will not evaluate the adequacy of the consideration. (D) is incorrect because the parties' status as nonmerchants is irrelevant. While the Code relaxes the Statute of Frauds rule in the case of a written confirmation between merchants [U.C.C. §2-201(2)], that exception does not apply here. Thus, even if the parties were merchants, the agreement would be unenforceable.

Question 25 An environmentalist divided her 25-acre property into 100 quarter-acre residential lots. At the time the environmentalist sold her lots, there was a recycling center about one mile from the western boundary of the development. She included in the deed of all 100 grantees the following provision: Grantee covenants for herself and her heirs and assigns that all aluminum cans, glass bottles, and grass clippings of Grantee and her heirs and assigns shall be recycled. This covenant runs with the land and shall remain in effect as long as there is a recycling center within five statute miles of the development. A buyer purchased a lot in the development. Her deed contained the recycling clause. Two years later, the buyer decided to give the property to her niece as a gift. The niece's deed to the property contained the recycling covenant. Shortly after the niece took possession of the house, the recycling center moved its location to a new site about four and a half miles from the development. When the niece put the house up for sale, she said nothing to prospective buyers about recycling. The house was purchased by a veteran who had lost the use of his legs. The veteran's deed did not contain the recycling clause, and he hired a local disposal service to carry away his garbage and a landscaper to maintain the yard. The landscaper bagged the grass clippings and they were removed by the disposal service, which put all the trash and clippings in a landfill. When the veteran's neighbors informed him of his duty to recycle, he told them that he knew nothing of the covenant and that it would be difficult for a person in his physical condition to haul cans, bottles, and clippings to the recycling center. Unfazed, the neighbors filed suit to require the veteran to comply with the covenant or pay damages. The veteran's best defense is which of the following? (A) The veteran's deed did not contain the covenant. (B) The covenant does not touch and concern the land. (C) An intelligent inspection of the neighborhood would raise no inference that the covenant existed. (D) The veteran's physical condition requires a balancing of hardships by the court.

25. Real Property Answer to Question 25 (B) The veteran's best defense is that the covenant does not clearly "touch and concern" the land. While recycling may benefit the community at large, "touch and concern" involves the relationship between landowners at law. Recycling by the veteran does not directly benefit the other landowners in the use and enjoyment of their land. Thus, (B) is correct. (A) is wrong because even though the veteran's deed does not contain the covenant, he has record notice because the restriction is in his chain of title. (C) is wrong because servitudes implied from a common scheme apply only to negative covenants, and the recycling requirement is an affirmative covenant. Thus, this defense does not go to the point. (D) is wrong because it goes only to issues in equity. The suit includes a claim for damages at law. In any case, balancing of hardships is not generally applied in such cases (although some courts might elect to do so).

Question 26 A plaintiff brought suit against a manufacturing company, seeking to recover for damages he suffered when his car's engine burst into flames following the use of an engine additive made by the company. The plaintiff contends that the manufacturing company was negligent and in breach of warranty. An automobile engineer sat in court while the plaintiff testified to the events concerning the engine fire. The plaintiff's testimony was not challenged or rebutted. The plaintiff calls the engineer to the stand and asks him whether, based on the plaintiff's prior testimony, it was possible for a car engine to burst into flames as it did. The engineer's testimony would be: (A) Inadmissible, because the engineer's opinion was not elicited by means of a hypothetical question. (B) Inadmissible, because the engineer was in the court while the plaintiff testified concerning the engine fire. (C) Admissible, because the engineer was in the court while the plaintiff testified concerning the engine fire. (D) Admissible, as long as the engineer's opinion is based only on admissible evidence.

26. Evidence Answer to Question 26 (C) The engineer's testimony is admissible because it is based on knowledge gained by him at trial. Facts or data upon which expert opinions are based may be derived from presentation at trial. One acceptable method of doing this is to have the expert attend the trial and hear testimony establishing the facts. Thus, (B) is incorrect. (A) is incorrect. Under the Federal Rules, a hypothetical question is not required to elicit an expert's opinion. (D) is incorrect because an expert may also base his opinion on facts supplied to him outside the courtroom, including types of facts not admissible into evidence, as long as they are reasonably relied on by experts in the field.

Question 27 A state statute prohibits speechmaking and loud public gatherings within 250 feet of the state's legislative chamber when the legislature is in session, but permits silent picketing at any time, as long as the picketing does not interfere with pedestrians or traffic. The nearest place to the legislative chamber where speeches could be made during a session is a large public park directly opposite the chamber. During a controversial debate on a proposed bill to ban abortions, a man in the park began voicing his support of the ban. As the man spoke, a crowd of about 250 gathered. When fervor built, the man urged the crowd to cross the street with him to the steps of the legislative chamber to make their voices heard within the legislature. When the chanting crowd reached the front of the chamber, the state police dispersed the crowd and arrested the man, who was subsequently charged with violating the statute. If the man challenges the constitutionality of the statute under which he was charged, a court will most likely find the statute: (A) Constitutional on its face and as applied to the man. (B) Constitutional on its face but not as applied to the man. (C) Unconstitutional on its face, because a state's citizens have a right to take their complaints to their state legislature. (D) Unconstitutional on its face, because it permits silent picketing while prohibiting other picketing.

27. Constitutional Law Answer to Question 27 (A) The statute is likely to be held constitutional on its face and as applied. The First Amendment protects the freedom of speech. Generally, content-based restrictions on speech will be found to violate the First Amendment. However, government may place reasonable restrictions on the time, place, and manner of speech. A speech regulation of a public forum will be found reasonable only if it is content‑neutral, it is narrowly tailored to serve an important government interest, and it leaves open alternative channels of communication. The regulation here passes muster. The area within 250 feet of the legislative chamber probably is a public forum. However, the regulation seems reasonable. It prohibits all loud public gatherings near the chamber when the legislature is in session to vote or debate. Thus, the regulation does not differentiate speech based on content. Moreover, the ban extends only to loud gatherings close to the chamber while the legislature is working. It appears to have the important purpose of enabling legislators to hear one another, and it seems to be narrowly tailored to that purpose. Moreover, the regulation leaves other channels of communication open (such as silent picketing). The application of the statute to the man does not make it unconstitutional as applied; there is no reason why he could not have spoken as noisily as he wanted to in the park. Thus, (B) is incorrect. (C) is incorrect because there are many methods to direct complaints to a legislator without yelling through his window. (D) is incorrect because silent picketing can be considered to be reasonable at the same time that loud picketing is prohibited.

Question 28 A restaurant owner properly sued a food supplier in federal district court for breach of contract and timely demanded a jury trial. The complaint asserted both legal and equitable claims. Which of the following statements correctly states the proper order for trying both claims? (A) All legal claims should be tried first by the jury. (B) All equitable claims should be tried first by the court. (C) Legal and equitable claims may not be tried together, so the order does not matter since there will be separate trials. (D) It is up to the federal district court judge's discretion which claim will be tried first.

28. Civil Procedure Answer to Question 28 (A) If legal and equitable claims are joined in one action involving common fact issues, the legal claim should be tried first to the jury and then the equitable claim to the court (the jury's finding on fact issues will bind the court in the equitable claim). (B) is wrong, because it misstates the rule. As stated above, the legal claim will first be tried by the jury followed by the equitable claim. (C) is wrong because legal and equitable claims can be consolidated together when the actions have a common question of law and fact. (D) is wrong because the Supreme Court has held that if legal and equitable claims are joined in one action involving common fact issues, the legal claim should be tried first.

Question 29 A patient went to a highly reputable dentist, who honestly told her that she needed a lot of dental work, some of it involving complex procedures. When she asked the dentist what the cost would be, he told her "about $3,500." The patient agreed to use him as her dentist and he began her treatment. When the patient's treatment was finished, the dentist sent her a bill for $4,100, explaining that the higher bill was because more expensive inlays were used, and that he carefully documented the cost of his materials and had sound medical reasons for his decision. The patient honestly believed that it was unfair for the dentist to charge her $4,100. Therefore, she sent the dentist's invoice back to him, along with a check for $3,500. On the check the patient had clearly written "Payment In Full." The dentist read the notation on the check and deposited it at his bank. The dentist made no notation of his own on the check other than his signature on the back as an indorsement. Two weeks later the dentist called his bank to make sure that the patient's check had cleared. He then immediately filed suit against the patient for $600. Will the court award the dentist $600? (A) Yes, because the dentist merely estimated the cost of the dental work to be $3,500. (B) Yes, because the dentist can document that the precious metal inlays were medically necessary and that he charged a fair price for them. (C) No, because there has been an accord and satisfaction of the original debt. (D) No, because there is an account stated.

29. Contracts Answer to Question 29 (C) The dentist will not prevail because the contractual duty was discharged by an accord and satisfaction. An accord is an agreement in which one party to an existing contract agrees to accept, in lieu of the performance that she is supposed to receive from the other party, some other, different performance. Satisfaction is the performance of the accord agreement. The effect of this performance is to discharge both the accord agreement and the original contract. An accord and satisfaction may be accomplished by a good faith tender and acceptance of a check conspicuously marked "payment in full" where there is a bona fide dispute as to the amount owed. The dentist and the patient agreed that the dentist would perform the dental work needed by the patient in return for the patient's promise to pay "about $3,500." The facts indicate that, following the actual dental work, there ensued a good faith dispute as to whether the patient owed only $3,500 or the additional $600 as well. The patient in good faith tendered to the dentist a check marked "payment in full," which notation the dentist saw before he deposited the check into his account. The dentist's acceptance of the patient's check gives rise to an accord and satisfaction with regard to the disputed original debt. Thus, the dentist is deemed to have accepted the amount tendered by the patient as full payment for the dental services performed, and the patient's duty to pay for the services is discharged. (A) is incorrect because, even if the dentist could have successfully asserted that $4,100 was within the range of the amount "about $3,500," his actions with respect to the check constitute an acceptance of the amount tendered therein. (B) is incorrect because, with the existence of an accord and satisfaction, the patient's duty to pay is deemed to be discharged. Thus, it is irrelevant at this point that the inlays may have been medically necessary or that the dentist charged a fair price for them. (D) is incorrect because an account stated is a contract whereby parties agree to an amount as a final balance due. For an agreement to be an account stated, there must be more than one prior transaction between the parties. The dispute between the patient and dentist involves only one transaction. Thus, there is no account stated.

Question 3 A woman decided to have a painting done of herself. She contracted in writing with an artist, who agreed to paint the woman for $10,000. The fee was payable on completion of the painting, provided that the painting was to the woman's "complete and utter satisfaction." On the same afternoon that the artist entered into the contract with the woman, he assigned the contract to his cousin. The artist then painted the woman's picture. After the job was done, the woman told him, "That's a very good likeness of me, but it shows my defects, so I'm not satisfied." She refused to accept the painting or to pay the artist or his cousin. Can the cousin recover from the woman? (A) Yes, because the condition in the agreement between the woman and the artist did not apply to his cousin. (B) Yes, because otherwise an unjust enrichment will occur. (C) No, because rights arising under personal services contracts are not assignable. (D) No, because the woman was not satisfied with the painting.

3. Contracts Answer to Question 3 (D) The cousin will not recover from the woman because she has a defense inherent in the contract. When one of the original parties to a valid contract assigns his rights under the contract to a third party, the assignee may enforce his rights against the obligor directly but is generally subject to any defenses that the obligor had against the assignor. As long as the defense is inherent in the contract, such as failure of a condition, it is always available against an assignee because it was in existence when the contract was made (even if whether the obligor would be able to utilize it was uncertain). Here, the artist (the assignor) and the woman (the obligor) had a valid contract—her promise to purchase the painting only if she was satisfied with it is not illusory because she has to exercise her right of rejection in good faith. When the artist assigned his rights under the contract to his cousin (the assignee), his cousin became subject to the condition in the contract that the woman be satisfied with the painting. Her dissatisfaction with the painting excuses her duty to pay for it; this is a defense inherent in the contract that precludes the cousin's recovering from the woman. (A) is incorrect because the assignee always takes subject to conditions in the original agreement between the obligor and the obligee. The only defenses that the obligor could not raise against the assignee are setoffs and counterclaims unrelated to the assigned contract that came into existence after the obligor learned that the contract was assigned. (B) is incorrect because the woman has not been enriched by the artist's services. She has justifiably refused to accept the painting and has received no benefit from the transaction that would constitute unjust enrichment. (C) is incorrect because the only right that the artist has assigned is the right to receive payment from the woman if she accepted the painting. The woman's duty is the same regardless of to whom she has to pay the money; therefore, the artist could validly assign his right to his cousin. Note that the analysis would be different if the artist had also attempted to delegate his duty of painting the woman to his cousin: duties involving personal judgment and skill may not be delegated. When an assignor assigns "the contract," the words are interpreted as including a delegation of the duties unless a contrary intention appears. Here, the contrary intention is indicated by the fact that the artist did the painting rather than his cousin; hence, there was no attempt by the artist to delegate a nondelegable duty.

Question 30 A landowner leased 150 acres of farmland to a produce company for 15 years. The produce company used the land for crops along with several other contiguous acres that it owned or leased. About four years into the lease, the state condemned a portion of the leased property because it intended to build a highway. As a result, too little property remained for the produce company to profitably farm, although there still existed the farmhouse on the property, which was being used by one of its foremen. The produce company gave the landowner 30 days' written notice that it considered the lease to have been terminated because of the condemnation. In a suit for breach of contract, the landowner would probably: (A) Lose, because the condemnation made it economically undesirable for the produce company to continue to lease the property. (B) Lose, because when there is a condemnation, the tenant's obligation to pay rent is extinguished. (C) Win, because the produce company can still use the farmhouse, and the rental value would be adjusted accordingly. (D) Win, because the condemnation did not affect the produce company's obligation to pay the full rental price, although it is entitled to share in the condemnation award.

30. Real Property Answer to Question 30 (D) The landowner probably will win in a breach of contract suit. In partial condemnation cases, the landlord-tenant relationship continues, as does the tenant's obligation to pay the entire rent for the remaining period of the lease. The tenant is, however, entitled to share in the condemnation award to the extent that the condemnation affected the tenant's rights under the lease. Therefore, (B) and (C) are incorrect. (A) is not correct because the law of landlord and tenant traditionally refuses to recognize frustration of purpose as grounds for termination of a lease.

Question 5 A landowner conveyed 30 acres of property "To my brother, his heirs and assigns, so long as the premises are used for agricultural purposes, then to my cousin, his heirs and assigns." As a consequence of the conveyance, the landowner's interest in the 30 acres is: (A) A right of entry. (B) A possibility of reverter. (C) A fee simple absolute because the conveyance violates the Rule Against Perpetuities. (D) Nothing.

5. Real Property Answer to Question 5 (B) The landowner's interest in the 30 acres is a possibility of reverter. Under the Rule Against Perpetuities, the attempt to give the cousin an executory interest is void because his interest could vest more than 21 years after a life in being. The courts will strike the gift over to the cousin and will then read the rest of the conveyance as it stands. Thus, (C) is incorrect. The "so long as" language creates a fee simple determinable in the brother, meaning that the grantor retains a possibility of reverter. Thus, (A) and (D) are incorrect.

Question 31 A man parked his car on a city street and entered a local tavern. While the man was inside, vandals broke the ignition motor on his car and smashed the taillights. Later that night the man left the tavern and attempted to start his car, but was unable to because of the vandals' actions, which he had not been aware of. Police in a patrolling squad car noticed him entering the car and attempting to start it. When the man got out of the car, the police approached him and asked him to identify himself. When he refused, he was arrested and charged with attempting to drive an automobile at night without functioning taillights. The man's best defense is: (A) Entrapment. (B) Legal impossibility. (C) Factual impossibility. (D) Lack of requisite intent.

31. Criminal Law Answer to Question 31 (D) The man's best defense is lack of intent. The crime of attempt is a specific intent crime, requiring the specific intent to carry out the crime in question. Since the man did not know that vandals had smashed his taillights, he never intended to drive an automobile at night without functioning taillights. (A) is incorrect because he did not attempt to drive the automobile at the instigation of the police. (B) is incorrect because legal impossibility is not applicable to these facts. (C) is incorrect because factual impossibility is not a good defense to a charge of attempt.

Question 32 While driving in a city in State A, a citizen of State B struck a pedestrian who was a citizen of State C. The pedestrian sued both the driver and the State A city in federal district court, seeking $100,000. The pedestrian alleged that the driver was liable for negligently operating his car and that the State A city was liable for negligently maintaining a traffic signal. The driver, who owns an office supply wholesaler, also has a contract claim against the State A city for $80,000 worth of furniture that the city allegedly purchased and received but never paid for. Assume that State A has waived any applicable governmental immunity. If the driver files a cross-claim for negligence against the State A city to recover damages for his injuries in the accident with the pedestrian, can the driver join with that cross-claim his contract claim for the purchase price of the furniture and maintain the claim in the same federal action? (A) No, because defendants cannot add claims against co-defendants that are not related to the original claims asserted by the plaintiff. (B) No, because the court does not have supplemental jurisdiction over the driver's contract claim against the State A city. (C) Yes, the driver must assert the contract claim or he will be foreclosed from asserting it later. (D) Yes, the driver may join his contract claim with his negligence cross-claim against the State A city, but is not required to do so.

32. Civil Procedure Answer to Question 32 (A) The driver cannot join his contract claim with his negligence cross-claim. A party may assert a cross-claim against a co-party only if the cross-claim arises from the same transaction or occurrence as that of the original action or of a counterclaim. Here, the contract claim is unrelated to the pedestrian's negligence claim and therefore cannot be asserted in this case. (D) is therefore incorrect. (B) is incorrect because the court would have diversity jurisdiction over the contract case. (C) is incorrect because the contract claim is not a compulsory counterclaim.

Question 33 A landowner owned a large commercial building downtown and the vacant lot next to his building. The landowner agreed to let an inner-city Scout troop use the vacant lot to practice outdoor-type activities. One weekend while the landowner was away on business, the Scout leader asked the landowner's building manager if he would permit the Scouts to practice their archery. The building manager agreed, but insisted that the targets be set up against the building so that an errant shot would merely strike concrete and not injure someone on the streets bordering the lot. The building manager supervised the set-up of the targets and ensured that responsible adults were present to assist the Scouts, then left the area. One of the Scouts jokingly shot an arrow up into the sky, and it came down on the street next to the vacant lot, striking and injuring a motorcyclist who was riding past. If the motorcyclist does not prevail in an action against the landowner for personal injuries, it will be because: (A) The landowner had no personal knowledge of the archery practice by the Scouts. (B) The precautions taken by the landowner's building manager were those a reasonable person would have taken. (C) The Scout acted recklessly by shooting the arrow into the air. (D) The failure of the adults present to prevent the Scout's action was a superseding cause of the injury.

33. Torts Answer to Question 33 (B) If the landowner prevails, it will be because his building manager was not negligent. The landowner owed a duty of reasonable care to travelers on highways adjacent to his land, such as the motorcyclist. However, if the landowner's employee was not negligent, the landowner would not be liable. (A) is incorrect because, if the manager was negligent, the landowner's lack of knowledge would not prevent application of respondeat superior. (C) is incorrect because, even though the Scout may have acted recklessly, the landowner could be liable if the employee did not exercise due care. (D) is incorrect because the failure of the adults to stop the Scout, even if it were considered negligence, would not be so unforeseeable as to cut off any previous negligence by the landowner's employee. The better basis for the result is that the landowner's employee was not negligent.

Question 34 A plaintiff sued a defendant for negligence when the tractor that he was driving at a construction site collided with the plaintiff's car. The plaintiff alleged that she was driving in a proper lane when the tractor collided with her car. The plaintiff's counsel called the responding police officer to testify that the defendant's employee made a statement to the police officer, in the defendant's presence, that the defendant "accidentally went too far into traffic," and the defendant did not say anything. The trial judge should rule that this evidence is: (A) Admissible, because the employee was working for the defendant, and as his agent, could make a statement against the defendant's interest. (B) Admissible, because silence may be deemed an admission. (C) Inadmissible, because the employee's statement was hearsay, and the defendant's silence is also hearsay. (D) Inadmissible, because the employee has not yet testified to his statement.

34. Evidence Answer to Question 34 (B) The judge should rule the evidence admissible as an admission. Although it is arguable that a person who may be liable for negligence would reasonably remain silent when in the presence of a police officer, this is the best answer because silence may be deemed an admission in a situation in which a reasonable person would have responded to an accusation. (D) is wrong because there need be no foundation evidence. (C) is wrong; the silence is deemed the adoption of the statement. Thus, the employee's statement would be an admission. (A) is wrong because, even if the employee is the defendant's agent, the statement against interest exception to the hearsay rule requires the unavailability of the declarant (i.e., the defendant here).

Question 35 In a claim for damages in a personal injury action, a plaintiff's attorney sought to introduce evidence of the plaintiff's testimony made to her boyfriend several days after her accident that "I must have sprained my neck when it happened because it hurts so much." The plaintiff is also planning to offer medical evidence that her neck was sprained. The judge should rule the testimony: (A) Inadmissible, because it is hearsay not within any exception. (B) Inadmissible, because the plaintiff is not qualified to give testimony as to her medical condition. (C) Admissible, because the plaintiff is also going to present medical evidence that her neck was sprained. (D) Admissible, to show that the plaintiff had suffered physical pain.

35. Evidence Answer to Question 35 (D) The judge should rule that the plaintiff's testimony is admissible. Although it was hearsay, the plaintiff's testimony was to show she was suffering pain, and is an exception to the hearsay rule as a declaration of present physical sensation. Statements of symptoms being experienced, including the existence of pain, are admissible under the Federal Rules, even if not made to a doctor or other medical personnel. Thus, (A) is wrong. (B) is wrong because the plaintiff's testimony is not to establish that she suffered a "sprained" neck, which would require an expert witness, but just to establish that her neck was in pain. (C) is wrong because the plaintiff's statement would be admissible to show her current physical condition even if she had not planned to introduce medical evidence.

Question 36 A defendant, on trial for robbing the victim of some jewelry, relied on the defense that he was only trying to recover property that the alleged victim had previously stolen from him. The trial court instructed the jury that the prosecution must prove guilt beyond a reasonable doubt, and that if the jury should find that the defendant had established by a preponderance of the evidence that he was only trying to recover his property, they should find him not guilty. After he was convicted of robbery, the defendant asserts that the instruction to the jury was error. His conviction should probably be: (A) Reversed, because the defendant need only convince the jury of any defense to a reasonable certainty, not by a preponderance of the evidence. (B) Reversed, because the instruction put a burden on the defendant that denied him due process of law. (C) Affirmed, because the defendant's burden to show that he was trying to recover his property was not one of ultimate persuasion, but only to produce evidence to rebut the legitimate presumption that the robbery was conducted with the intent to permanently deprive the victim of the jewelry. (D) Affirmed, because the instruction was an accurate statement of the law.

36. Criminal Law Answer to Question 36 (B) The defendant's conviction should be reversed. Robbery requires an intent to permanently deprive the victim of her property. An intent to recover property that the defendant believes is his would not be a sufficient intent. The prosecution must prove every element of the crime beyond a reasonable doubt, and putting the burden of persuasion to show an innocent intent on the defendant would deprive him of due process of law, since it would relieve the prosecution of its burden to show the required intent for robbery. Thus, the instruction was improper and (D) is wrong. (A) and (C) are wrong. The defendant does not carry any burden of proof with respect to an element of the crime. The burden is on the prosecution to prove each element beyond a reasonable doubt.

Question 37 A defendant is on trial for arson of a restaurant. Chemical tests by the fire department indicate that gasoline was used as the igniting agent of the fire. The prosecution calls to the stand a waitress who works at a diner near the burned restaurant. She will testify that on the night of the fire, the defendant came into the diner smelling like gasoline. Should the court admit this testimony over the defendant's objection? (A) No, it is inadmissible as the opinion of a nonexpert witness. (B) No, it is inadmissible because the best evidence is the result of the chemical tests. (C) Yes, it is admissible lay opinion testimony. (D) Yes, it is admissible expert testimony because everyone who drives a car is an expert as to the smell of gasoline.

37. Evidence Answer to Question 37 (C) The witness should be allowed to testify as to what she perceived. To be admissible under the Federal Rules, evidence must be probative of a material issue in the case and must be competent (i.e., not otherwise excludable). Evidence is material if it relates to an issue in the case, and it is probative if it tends to prove the fact for which it is offered. Evidence is competent if it does not violate a specific exclusionary rule. At issue here is whether the defendant started the restaurant fire. If gasoline was used to start the fire, the fact that the defendant was seen near the fire and smelled like gasoline makes it more likely that he started the fire, so the proffered evidence is material and relevant. It is also competent; under the Federal Rules, opinion testimony by lay witnesses is admissible when (i) it is rationally based on the perception of the witness, and (ii) it is helpful to a clear understanding of her testimony or to the determination of a fact in issue. Matters involving sense recognition, such as what something smelled like, are common subjects of opinion testimony. Here, the witness's testimony satisfies both elements of the test and should be admitted. Thus, (C) is correct and (A) is incorrect. (D) is incorrect because to testify as an expert one must have special knowledge, skill, experience, or education as to the subject of her testimony, and if everyone who drives a car knows the smell of gasoline, there is nothing special about that knowledge. (B) is incorrect because the best evidence rule requires that the original document be produced only when the terms of the document are material and sought to be proved. The fact to be proved—that gasoline was used—exists independently of any written record of the chemical tests; therefore, the best evidence rule does not apply.

Question 38 A patient sought psychiatric treatment from a psychiatrist. During the treatment, the psychiatrist, unbeknownst to the patient, videotaped her. No sound recording was made of the sessions, but the psychiatrist was conducting a study on "body language" and planned to use the videotapes in those experiments. The patient learned that the psychiatrist had been videotaping their analysis sessions and brought an action against him for battery. If the patient does not prevail as to this theory, it will probably be because: (A) She did not suffer any injury as a result of the psychiatrist's actions. (B) The psychiatrist had an implied consent to take the actions he did as part of the patient-physician privilege. (C) She did not suffer an offensive touching. (D) The psychiatrist intended that his actions would foster medical research.

38. Torts Answer to Question 38 (C) The patient cannot make out a case for battery because she did not experience an offensive touching. To make out a prima facie case for battery, the plaintiff must show an intentional act by the defendant that caused harmful or offensive contact to the plaintiff's person. Here, nothing indicates that the patient was touched in any way; thus, (C) is correct. (A) is incorrect because injury is not an element of battery; battery can be established even absent a showing of injury or damages. (B) is incorrect because if the elements of battery were present, the physician-patient privilege would not protect the psychiatrist here because the patient did not impliedly consent to the taping. When a patient consents to a doctor's treatment, she impliedly consents to all necessary touching that goes along with the treatment. However, when the doctor goes beyond the scope of the acts consented to and does something substantially different, the defense of consent is no longer available. Here, the patient consented to talk with the psychiatrist, but that is substantially different from agreeing to be the subject of an experiment. Thus, if taping the patient were a battery, the psychiatrist could not rely on implied consent to relieve him of liability. (D) is incorrect because a benevolent motive is not a defense to a battery. If the other elements of battery were present, the fact that the battery occurred to foster medical research would not be a defense—that would allow psychiatrists to physically strike their patients merely to study their reactions!

Question 39 A defendant is on trial in federal court for the armed robbery of a casino. The defendant claims that he was out of town at the time of the robbery. The defendant calls an alibi witness to the stand to testify that she was with him on the trip. When asked where she was and who she was with on the date in question, the witness stated that she could not recall. She said she recalls spending a weekend at a bed and breakfast this spring, but she does not recall the date or her traveling companion. The defendant's attorney then showed the witness a letter written by her on stationery from the bed and breakfast, and asks her to look at it and try to answer the question again. The prosecution objects. The objection should be: (A) Overruled, because this is a past recollection recorded. (B) Overruled, but the witness cannot depend on the terms of the letter when answering. (C) Sustained, because the letter is hearsay. (D) Sustained, because the letter has not been properly authenticated.

39. Evidence Answer to Question 39 (B) The prosecution's objection should be overruled. If a witness's memory is incomplete, the examiner may seek to refresh her memory by allowing her to refer to a writing or anything else—provided she then testifies from present recollection and does not rely on the writing. (A) is incorrect because past recollection recorded is a hearsay exception that applies when a party is seeking to introduce a particular kind of writing. Here, the defendant is not seeking to introduce the writing; he merely wants the witness to look at it. Thus, (A) is incorrect. (C) and (D) are incorrect because the letter is not being offered into evidence. Hence, the letter is not hearsay and does not need to be authenticated.

Question 4 A company that provided electrical and communication services to industries determined that its main trunk line of copper conduit needed to be replaced. The line ran through an industrial park, and the company had access to it by an easement. At the end of the workday on Friday, the copper conduit that had not yet been disconnected was left exposed in the trench. In accordance with the company's policy, intended to discourage theft of the copper, the conduit was still electrically charged. The next morning, a man was walking by the industrial park and saw the construction site. Ignoring the "no trespassing" signs, he came up to the trench and saw the copper conduit, which he decided to try to steal. He climbed over the barriers and climbed partway into the trench to try to pull out some of the conduit. He received an electric shock as he made contact with the conduit, causing him to fall into the trench and suffer severe electrical burns. If the man sues the company for his injuries, is he likely to prevail? (A) Yes, because the company used unreasonable force to protect its property. (B) Yes, because force applied by mechanical devices may not be used to protect property alone. (C) No, because the company owed no duty to a trespasser. (D) No, because the man assumed the risk.

4. Torts Answer to Question 4 (A) The man will prevail because the company used unreasonable force to protect its property. One may use reasonable force to prevent the commission of a tort against one's property, real or personal. However, force that will cause death or serious bodily harm may not be used. In addition, indirect deadly force may not be used when such force could not lawfully be directly used. Here, the company kept the power running to prevent the theft of its copper. In effect, this amounts to the use of indirect deadly force as a means of preventing a tort to personal property. As explained above, use of such force to protect property is not permitted. Thus, the man will prevail in his suit against the company. (B) is incorrect because it is too broad. Force may be used to protect only property if such force is reasonable, regardless of whether it is directly applied or indirectly applied by mechanical devices. The problem here is that the force used by the company was unreasonable because it was deadly force. Regarding (C), it is true that an owner or occupier of land owes no duty to an undiscovered trespasser. However, the company is not an owner or occupier of the land on which the line runs, but simply the owner of the line itself and the holder of an easement across the land. Thus, the company is held to the general duty of due care regardless of the man's status, rather than the more limited duty owed by a landowner or occupier to a trespasser. (D) is incorrect because the facts do not indicate that the man assumed any risk. To have assumed a risk, a plaintiff must have known of the risk and must have voluntarily gone ahead in the face of the risk. Here, the facts do not indicate that the man knew that electrical current was still running through the line. In fact, because it was under construction and exposed, the man likely assumed that the power was not running. Thus, he did not know of the risk and could not have voluntarily assumed such a risk. Note also that, while the question does not indicate the theory on which the man is basing his cause of action, it may be based on a theory of intentional tort (specifically battery), in which case assumption of risk would not be an appropriate defense.

Question 40 The owner of a semi-pro baseball team offered a former player a position as the team's manager. During negotiations, the owner agreed to pay the manager $300 per week, but insisted that if the manager quit during the season, he would have to pay a "penalty" of $200 per week for each week that he did not manage the team, because it would cost the owner several hundred dollars to replace him during the season. The parties agreed in writing to those terms. The manager managed the team for 14 weeks with only mixed success. Nevertheless, with 10 weeks left in the season, he was offered and accepted a job as manager of a professional baseball team. Fortunately for the owner of the semi-pro team, the manager's replacement had great success with the team, causing attendance to skyrocket. If the owner of the semi-pro team brings suit against the manager to recover $2,000, the amount due under the "penalty" provision, the owner will: (A) Not prevail, because "penalty" clauses in contracts are not enforceable. (B) Not prevail, because the owner of the semi-pro team was not harmed by his breach. (C) Prevail, because the manager can be penalized for his willful breach. (D) Prevail, because the "penalty" provision is enforceable.

40. Contracts Answer to Question 40 (D) The owner of the semi-pro team will prevail. Although the $200 per week damages was denominated a "penalty" by the parties, in fact it operates as a reasonable liquidated damages clause. The parties to a contract may stipulate what damages are to be paid in the event of a breach if (i) damages are difficult to ascertain at the time the contract is formed, and (ii) the amount agreed on is a reasonable forecast of compensatory damages in the case of a breach. These conditions have been met here. (A) is wrong because while it is true that penalties will not be enforced, a court would not construe the provision here as a penalty since it meets the requirements above. (B) is irrelevant. (C) is a misstatement of the law.

Question 41 The manager of a monthly antique market was looking to hire a professional appraiser who would tell patrons, for a fee of $10 per item, what their antique is worth. Because the manager had had problems at other antique markets he had run because appraisers built up a popular following and then abruptly quit for a better job, he emphasized during the hiring interview the importance of honoring the contract to its completion. When the manager offered the job to an experienced appraiser, and the appraiser accepted, the written contract signed by the appraiser contained, in addition to an agreed-to salary, a liquidated damages clause for early termination of the contract. It also contained another clause providing that the appraiser would receive 5% of all gate receipts to be paid as a bonus at the end of the contract, which ran for one year. Eight months into the contract, the manager's worst fears were realized when the appraiser got a more lucrative offer and abruptly quit, leaving the manager scrambling to find a replacement for him. In response to the manager's suit for breach of contract, the appraiser brings a countersuit to recover 5% of gate receipts for the antique markets at which he worked. Will the appraiser be successful in his countersuit? (A) Yes, because his breach was not substantial. (B) Yes, because the manager's only remedy was the liquidated damages clause. (C) No, because working the entire year was an implied condition of the contract. (D) No, because working the entire year was an express condition of the contract.

41. Contracts Answer to Question 41 (C) The appraiser will not recover a percentage of the receipts from the manager. In construing the contract, the court will attempt to give effect to the reasonable expectations of the parties. Even though they did not expressly so provide, the parties probably intended the bonus to be incentive for the appraiser to stay and that it be paid only if he completed the term of the contract. Thus, such a condition will be implied. Therefore, (D) is wrong. (A) is contrary to the facts because there were four months left in the 12-month contract. (B) is wrong because the manager's remedy does not affect the appraiser's right to payment.

Question 42 A gas company offered a generous sum to a landowner in exchange for permission to run pipes over a 15-foot strip of the landowner's property to supply natural gas to a new housing development. The landowner granted the gas company and its "heirs and assigns" an easement over the 15-foot strip of property "for pipeline purposes." The easement was duly recorded. When the housing development was ready for utility services, the gas company laid the pipe across the 15-foot strip of property. Shortly after the gas company finished laying its pipe, the landowner planted a garden of trees and flowers on the 15-foot strip. For the next 10 years, the gas company supplied natural gas to the housing development through that pipe. When the gas company no longer needed the lines that ran across the property, it sold its rights in the easement to an oil company that wished to run a crude oil pipeline across the same strip of property. Now, although the oil company confined its activities to the strip, it drove trucks onto the land, uprooted the trees, and tore up the flower garden in order to excavate for the laying of a larger pipe than the gas company had installed. The landowner filed suit to enjoin the oil company from further activities on the 15-foot strip and for damages for the destruction wrought on his property. Is the landowner likely to succeed in this suit? (A) Yes, because the gas company's sale of the easement to the oil company was without the landowner's permission. (B) No, because the gas company had the right to assign its interest. (C) No, as to the injunction, because the oil company is using the land for pipeline purposes, but the landowner is entitled to damages. (D) No, as to the injunction and damages, because the use is for pipeline purposes and this obviously implies the right to excavate in order to lay pipe.

42. Real Property Answer to Question 42 (C) The landowner will be unable to obtain an injunction but may recover damages against the oil company. An easement in gross that is commercial may always be transferred, and the permission of the holder of the servient estate is not needed. Thus, (A) is wrong. Because the oil company was within its rights under the terms of the easement, the laying of pipe is permissible and no injunction will lie. It is, however, the obligation of the holder of the easement to reasonably restore the surface following the excavation. Thus, (C) is correct, and (B) and (D) are wrong.

Question 43 A construction company was employed by the city to repair a broken pipe under a main street of the city. As part of its work, the construction company dug a deep trench down the center of the street. Warning signs and flashing lights were positioned at both ends of the trench to warn cars of the danger, but, except for two unlit warning signs along the sides, no barriers of any kind were put up on either side. A college student was walking home from the bus depot along the same street after dark. A heavy rain started, and in an effort to avoid getting more drenched, the student jaywalked instead of going to the corner to cross the street, which was empty of traffic at the time. In the dark and the heavy rain, the student did not realize that the road was dug up where he was crossing and fell into the trench. The student was knocked out when he hit the bottom of the trench and drowned when the water from the rain gathered at the bottom. The student's mother brought a wrongful death action against the construction company for the death of her son. The jurisdiction retains traditional contributory negligence rules and has a statute that makes it a misdemeanor for anyone to cross a main street such as this at any place except in the marked crosswalks. The construction company is likely to: (A) Prevail, because the student was negligent per se, and thus at fault for his own death. (B) Prevail, because it had marked off its trench with warning signs. (C) Not prevail, because a construction company is strictly liable for inherently dangerous conditions. (D) Not prevail, because it left an open trench unprotected.

43. Torts Answer to Question 43 (D) The construction company probably will not prevail in the wrongful death action against it. An excavator near a public road has a duty of due care to protect users of the road from straying and falling in. This duty is satisfied when the excavator has done everything reasonable to protect the open excavation by putting up barriers that are likely to prevent such accidents. The construction company may have reasonably protected cars driving on the street from falling in the trench, but no real step was taken to prevent pedestrians from falling into the trench at nighttime. Hence, (B) is wrong. (A) is wrong because, although the student may have violated the jaywalking statute, the statute is not likely to be applied here because this type of statute was designed to prevent vehicle-pedestrian accidents rather than falling into a trench. In addition, the facts do not establish any other conduct on the part of the student that would constitute contributory negligence. (C) is not the best answer because all that is required with regard to artificial conditions is that the excavator exercise due care to warn persons of the danger.

Question 44 A husband who believed that his wife was having an affair with his brother hired an arsonist to burn down the brother's house. They planned for the husband to take his brother to a ballgame so that the arsonist would be able to set the house on fire without detection. After the husband and brother left for the ballgame, however, the arsonist decided to abandon the plan and immediately left town without doing anything further. When the husband returned from the ballgame with the brother, he saw the house still standing and blurted out what was supposed to have happened. The husband and the arsonist were arrested and charged with conspiracy to commit arson. At the arsonist's trial, his attorney argued that he was innocent of the conspiracy because he decided not to go ahead with the plan, and nothing criminal had in fact occurred. At common law, a jury should find the arsonist: (A) Not guilty of conspiracy, because going to a ballgame is not a criminal overt act. (B) Not guilty of conspiracy, because the husband, not the arsonist, committed the overt act. (C) Guilty, because the husband executed his part of the plan. (D) Guilty, because the arsonist agreed to set the brother's house on fire.

44. Criminal Law Answer to Question 44 (D) The arsonist should be found guilty. A conspiracy is a combination or agreement between two or more persons to accomplish some criminal or unlawful purpose, or to accomplish a lawful act by unlawful means. The mens rea required for conspiracy is specific intent, in that both parties must intend to agree to accomplish some criminal or unlawful purpose. Thus, once the arsonist was hired by the husband and they came up with a plan to burn down the brother's house, the crime of conspiracy was completed. (C) is incorrect because it implies that carrying out the plan by at least one party is required; the conspiracy was complete even before the husband fulfilled his duties under the plan. Note that, while most states now require an overt act for conspiracy, the common law version does not. (A) is incorrect. Even if an overt act were required, it need not be in and of itself criminal. (B) is also incorrect. If an overt act were required, it need only be performed by one of the co-conspirators, not necessarily the conspirator on trial.

Question 45 Congress enacted legislation providing that, among other things, "federal courts shall not order any public educational institution to establish athletic activities or to modify existing athletic activities on the grounds that such activities are not provided on an equal basis to both men and women." Which of the following is the strongest argument for the constitutionality of the federal legislation? (A) Congress provides financial support for public educational institutions and is therefore empowered to place conditions on the expenditure of federal funds. (B) Athletics involves tremendous amounts of money and the occasional use of interstate means of transportation, thus falling within the commerce power. (C) Under Article III, Congress may restrict the jurisdiction of the federal courts. (D) The Fourteenth Amendment authorizes Congress to define governmental conduct that violates the Equal Protection Clause

45. Constitutional Law Answer to Question 45 (C) The strongest argument in favor of the constitutionality of the legislation is that Congress is explicitly authorized to restrict the jurisdiction of the federal courts under Article III. (D) is incorrect because Congress cannot define conduct that will violate the Fourteenth Amendment's Equal Protection Clause; it may only enact laws to prevent or remedy violations of rights already recognized by the courts. [City of Boerne v. Flores (1997)] (B) is incorrect because the statute does not regulate interstate commerce, but rather limits the power of the courts to order certain remedies. Neither does the statute affect the expenditure of any federal educational funds, because it is directed at the courts. Therefore, (A) is incorrect.

Question 46 Congress adopted legislation prohibiting the federal courts from ordering busing as a remedy for past racial discrimination in a public school district. Which of the following is the strongest argument that the federal legislation is unconstitutional? (A) The courts, not Congress, have the primary responsibility for defining the minimum requirements of the Equal Protection Clause of the Fourteenth Amendment. (B) The Privileges or Immunities Clause of the Fourteenth Amendment prohibits Congress from limiting the forms of relief afforded by federal courts. (C) Congress cannot limit the authority of federal courts to hear and decide cases properly presented for decision. (D) The legislation unduly burdens interstate commerce.

46. Constitutional Law Answer to Question 46 (A) The strongest argument against the legislation's constitutionality is that Congress cannot usurp the courts' authority to define the scope of the Equal Protection Clause. If the Equal Protection Clause requires a remedy for past racial discrimination, then Congress could not constitutionally interfere with the fashioning of a judicial remedy to achieve constitutionally required conduct. (D) is wrong because Congress may burden interstate commerce, because it has very broad power on that subject. (C) is wrong because Congress may limit the jurisdiction of federal courts. (B) is wrong because the Privileges or Immunities Clause protects individual rights against infringement by state government; it does not limit the powers of Congress vis-à-vis the federal courts.

Question 47 A State A citizen and a State B citizen were in a car accident. The State B citizen filed a negligence action against the State A citizen in a State A state court, seeking $500,000 in damages. The State A citizen immediately filed a notice of removal in the state court and in the appropriate federal district court. If the State B citizen believes the removal is improper and that the action should properly remain in the State A state court, what should the State B citizen do? (A) The State B citizen does not need to do anything because the removal is improper and will not be recognized by the courts. (B) The State B citizen should file an objection to the removal in the state court. (C) The State B citizen should file in the federal court a motion to remand the action to state court. (D) The State B citizen has no recourse because, once an action is removed to federal court, the federal court will retain the case unless it lacks subject matter jurisdiction.

47. Civil Procedure Answer to Question 47 (C) The State B citizen should file a motion to remand. The filing of a notice of removal ends state court jurisdiction and accomplishes the removal. The plaintiff, however, may file a motion in the federal court asking that action be remanded to state court, either because it was not properly removed or because the federal court lacks subject matter jurisdiction. (A) is incorrect. Although the federal court must remand the case to state court when it lacks subject matter jurisdiction, not all federal courts will remand a case on their own initiative for procedural defects in removal. Thus, it is incumbent on the plaintiff to seek remand. Depending on the jurisdiction, the court may not be able to remand on its own initiative, and, even if the court does have the authority to remand but fails to do so, the defect in removal would not be jurisdictional. (B) is incorrect because the proper procedure is to file a motion for remand in the federal district court. The state court does not have the authority to remand a case back to itself. (D) is incorrect. A procedural defect in removal may warrant a remand.

Question 48 A buyer entered into a written contract with a farmer to purchase the farmer's dairy farm. The contract contained a provision that the farmer's "land and inventory are valued at $175,000." The contract also contained a provision that stated, "This contract represents the entire agreement between the parties. No other promises or representations have been made." In a fraud action against the farmer, the buyer alleges that he purchased the farm only because the farmer had assured him that the land and inventory were worth $350,000, when they were in fact worth only $175,000. The buyer seeks to testify that during negotiations the farmer had said repeatedly that the value of the land and the inventory was $350,000, but at the advice of the farmer's attorney, he was going to list the value at $175,000 in the contract for tax purposes. The farmer's attorney objects. The evidence is: (A) Inadmissible hearsay. (B) Inadmissible under the parol evidence rule. (C) Neither hearsay nor violative of the parol evidence rule. (D) Admissible hearsay, unaffected by the parol evidence rule.

48. Evidence Answer to Question 48 (C) The buyer's offered testimony should be admitted. The statement by the farmer is not hearsay, because it is not offered to prove that the statement was true; rather, it is evidence of words which are a fraudulent representation, and hence themselves actionable. Alternatively, the statement is an admission by the farmer and therefore nonhearsay under the Federal Rules. It is not within the parol evidence rule, because the plaintiff is not trying to prove the meaning of the contract, but rather is alleging fraud. The parol evidence rule does not bar admission of parol evidence to show that what appears to be a contractual obligation is, in fact, no obligation at all. Hence, parol evidence is admissible to attack a contract on the grounds of fraud. (A), (B), and (D) are therefore wrong.

Question 49 A landowner developed and built a large apartment house on property he owned. After the apartment house was completed, the landowner conveyed the building and the land to his son for life, remainder to his granddaughter; subject, however, to a bank's mortgage. The bank held a mortgage on the building for $17,000, payable in annual installments of $2,000 plus interest. The son began to manage the day-to-day maintenance of the building as well as to collect rents. Despite the son's best efforts, the vacancy factor reached an unanticipated high and the rental income was not sufficient to cover the annual mortgage payment. Who is responsible for the mortgage payments and in what amount? (A) The son pays all. (B) The granddaughter pays all. (C) The son pays the interest and the granddaughter pays the principal. (D) The granddaughter pays the interest and the son pays the principal.

49. Real Property Answer to Question 49 (C) The son is responsible for the interest payments on the mortgage and the granddaughter is responsible for the principal. The doctrine of waste governs the obligations between a life tenant and the holder of the remainder regarding the payment of a mortgage on the property. Under this doctrine, a life tenant is obligated to pay interest on any encumbrances on the land, but he does not have to pay anything on the principal of the debt; reversioners or remaindermen must pay the principal in order to protect their interests. (C) correctly states that rule, and (A), (B), and (D) are necessarily wrong.

Question 50 A driver took her car to a garage to have her squeaking brakes repaired. The garage was owned by a corporation in which she held shares of stock. Unfortunately, rather than fixing the brakes, the garage made them worse. As a result, the driver had a bad accident when her brakes failed as she was navigating a tight curve. She brought a negligence action against the garage as a corporate entity in federal court, asserting diversity jurisdiction. She filed suit one day before the statute of limitations expired and served the garage with process one day after the statute of limitations expired. Eight months later the driver decides to add a shareholder's derivative claim based on corporate misconduct, but the statute of limitations has long expired. May she add the shareholder's derivative claim? (A) No, because the statute of limitations on the derivative claim has run. (B) No, because the corporation was not served until after the statute of limitations had run. (C) No, because the other shareholders did not receive notice of the suit within the statute of limitations period. (D) Yes, because the original complaint and service of process in the first action were both timely.

50. Civil Procedure Answer to Question 50 (A) The driver may not amend her complaint. In federal court, an amendment to a complaint "relates back" to the date the complaint was filed when the new claim arises from the same transaction or occurrence as the original claim. Here, the new claim does not arise out of the same transaction. Thus, the driver may not amend her complaint to add the shareholder's derivative claim to the suit. (B) is incorrect because the notice to the defendant is required within the statute of limitations period plus the time allowed for service of process. (C) is incorrect. Notice in accordance with Rule 23 will be given to the shareholders sometime after suit is filed. (D) is incorrect because it ignores the same transaction or occurrence requirement.

Question 6 A man and woman agreed to burn down a neighbor's house in retribution for some wrong the neighbor allegedly committed against them. Both the man and woman were arrested shortly after they poured gasoline on the neighbor's front porch. The man revealed to the police that he participated in the plan to ensure that nothing bad would happen to the neighbor, and that he had made an anonymous telephone call to the police alerting them to the crime, which enabled the police to arrest him and the woman "in the act." The woman stated that she would not have participated if not for the man's encouragement. If charged with a conspiracy at common law to commit arson, the woman should be found: (A) Not guilty, because she was not predisposed to commit the crime but for the man's encouragement. (B) Not guilty, because the man did not intend to commit arson. (C) Guilty, because there was an agreement, and pouring gasoline on the front porch was sufficient for the overt act. (D) Guilty, because arson is not a specific intent crime.

6. Criminal Law Answer to Question 6 (B) The woman should be found not guilty of a conspiracy to commit arson. To be convicted of a conspiracy at common law, it must be shown that at least two persons agreed to achieve an unlawful objective. Having two or more persons is a necessary element of conspiracy under the traditional bilateral approach. Here, the facts indicate that the man did not intend to achieve the objective of the conspiracy—to burn the dwelling house of another. Thus, the woman cannot be guilty of conspiracy to commit arson. (C) is incorrect. The man feigned his agreement, making the answer factually inaccurate. (D) is also incorrect. Although it is true that arson is not a specific intent crime, conspiracy is a specific intent crime, in that the prosecution must show that the defendant intended to agree and intended to achieve the unlawful objective. Thus, the fact that the underlying crime is not a specific intent crime is irrelevant. (A) is incorrect. Even if the woman would not have committed the crime without the man's inducement, that is not a defense for the woman. A person cannot be entrapped by a private citizen.

Question 7 As it approached the runway at an airport, a plane struck electrical wires and crashed. An injured passenger filed an action in federal district court against the Federal Aviation Administration ("FAA") under the Federal Tort Claims Act, alleging that the FAA was negligent in operating runway lights and performing its air traffic control functions. The passenger seeks to join in that action a state law negligence claim against the utility company that erected the power lines, alleging negligent placement of the lines. Both claims seek $1 million for the passenger's injuries. The passenger is a citizen of State A. The utility company is a State A corporation and all its offices and operations are in State A. Does the federal court have subject matter jurisdiction over the passenger's claim against the utility company? (A) No, because the claim is a state law claim between two citizens of the same state. (B) No, because, while the two claims arise from the same event and same injuries, supplemental jurisdiction does not extend to claims against a different party. (C) Yes, because the court has supplemental jurisdiction over the passenger's claim against the utility company. (D) Yes, because a plaintiff may join in an action all claims that he has against a particular defendant.

7. Civil Procedure Answer to Question 7 (C) The court has subject matter jurisdiction. Federal question jurisdiction is available when the plaintiff, in his well-pleaded complaint, alleges a claim that arises under federal law. Here, the claim against the FAA arises under federal law because it was brought under the Federal Tort Claims Act. Once a claim satisfies the requirements for original federal subject matter jurisdiction, the court has discretion to exercise supplemental jurisdiction over related claims that derive from the same common nucleus of fact and are such that a plaintiff would ordinarily be expected to try them in a single judicial proceeding. Here, the claim for negligence against the utility company for negligent placement of the power lines (resulting in the accident) is derived from the same transaction or occurrence as the Federal Tort Claims Act claim against the FAA for negligent operation of the runway lights and the air traffic control functions (resulting in the accident). Thus, the claim against the utility company falls within the court's supplemental jurisdiction and (C) is correct. (A) is incorrect. Although it is true that the court does not have diversity of citizenship jurisdiction because the utility company and the passenger are from the same state, the answer fails to consider the applicability of supplemental jurisdiction. (B) is incorrect because supplemental jurisdiction may extend to claims by or against different parties. (D) is an incorrect statement of the law. Although it is true that the Federal Rules allow a party to join claims he has against a particular defendant in one action, it does not obviate the need for subject matter jurisdiction.

Question 8 A man purchased a large flat screen plasma television and decided to mount it on the ceiling over his bed. The manual that came with the product included detailed instructions and illustrations on how to mount the television on different types of walls, along with all the required hardware, but contained neither instructions nor warnings regarding mounting on the ceiling. The man carefully followed the wall-mounting instructions and was satisfied that it would hold. In fact, however, the mounting was not appropriate for ceilings. The next night, a woman who was the man's overnight guest was seriously injured when the television came loose and fell on the bed. Will the woman prevail in a suit against the company that manufactured the television? (A) Yes, because the manufacturer had a duty to include warnings for all potential placements of its product. (B) Yes, if the manufacturer knew that its television was sometimes mounted on ceilings rather than walls. (C) No, if the manufacturer's manual had all of the customary warnings for this type of product. (D) No, because the man was negligent in mounting the television on the ceiling.

8. Torts Answer to Question 8 (B) Knowledge on the part of the manufacturer that its television was being mounted on the ceiling would give rise to a duty to include in the manual warnings against the practice or detailed instructions on how to safely mount it. The television hardware and instructions were appropriate for its intended mounting on the wall. However, courts in a strict liability case require a commercial supplier to anticipate reasonably foreseeable uses even if they are misuses of the product. If the manufacturer knew that members of the public were sometimes mounting the television on the ceiling, marketing the product without including either warnings against the practice or appropriate hardware and instructions on how to safely do so made the product so defective as to be unreasonably dangerous if it were improperly mounted. Under a strict liability theory, the manufacturer is liable for supplying a defective product. As a guest of a purchaser of the product, the woman is a foreseeable plaintiff; thus, the manufacturer may be liable to her. The defective product actually and proximately caused the woman to suffer serious injuries. Therefore, the manufacturer is liable to the woman in a strict products liability action. (A) is incorrect because the facts do not establish that the manufacturer was under a duty to include the warnings in its manual. Such a duty would exist if the manufacturer knew (as (B) states) or should have known that the television was being mounted on ceilings. (C) is incorrect because industry custom does not conclusively establish the applicable standard of care in a given case (although such customs are admissible as evidence of the standard to be applied). Here, including only the customary warnings in an instruction manual may be violative of the appropriate standard of care (i.e., manufacturers of these televisions may be under a duty to add additional warnings in the manual). Thus, the statement set forth in (C) will not by itself mean that the manufacturer will prevail. (D) is incorrect because, even if the man should have known that the television should not have been mounted on the ceiling, such negligence would not be a superseding cause of the injury because it would be ordinary foreseeable negligence. Consequently, the manufacturer would not be relieved of liability for the results of its own wrongful conduct, but would be held liable along with the man.

Question 9 A developer subdivided a 25-acre tract of land into 100 quarter-acre lots. On each lot she built a two-unit townhouse. The deeds to each of the purchasers contained a covenant that "the grantee, his heirs and assigns" would use the property only for single-family use. All deeds were promptly and properly recorded. Subsequently, the zoning laws were amended to allow multifamily use within the subdivision. Six months later, a social worker offered to purchase an original owner's unit that was for sale. The social worker informed the owner that she planned to operate a halfway house out of the unit, an activity in conformity with the applicable zoning regulations. Therefore, the owner did not include the single-family restriction in the deed to the social worker. If a neighbor, who purchased his lot from the developer, seeks to enjoin the operation of the halfway house, will he succeed? (A) No, because the deed from the owner to the social worker did not refer to the covenant. (B) No, because the social worker relied on the zoning regulations when purchasing the unit. (C) Yes, because the social worker had notice of the restrictive covenant. (D) Yes, but only if the neighbor can establish a common scheme for development.

9. Real Property Answer to Question 9 (C) The neighbor will succeed in enjoining the operation of the halfway house because the social worker had notice of the restrictive covenant. A covenant runs with the land to a subsequent purchaser with notice of the covenant if it touches and concerns the land and is intended to run. Notice may be actual or constructive. Here, the social worker was on record notice of the covenant because the original owner's deed was recorded. Restricting land to single-family use touches and concerns the land, and it is evident that the developer and the original owners, including the neighbor, intended it to run with the land by use of the language "grantee, his heirs and assigns." The social worker thus will be bound even though her deed did not refer to the covenant. Thus, (A) is incorrect. (B) is incorrect because compliance with zoning regulations does not excuse noncompliance with an enforceable covenant; both must be complied with. (D) is incorrect because the neighbor can prevail without needing to show a servitude implied from a common scheme, which comes into play when a developer subdivides land into several parcels and some of the deeds contain negative covenants and some do not. Here, the covenant relating to single-family use was in all of the original deeds and, as discussed above, it runs with the land. A covenant that runs with the land may be enforced as an equitable servitude if the assignees of the burdened land have notice of the covenant; the usual remedy is an injunction. Here, the social worker had record notice of the covenant and it runs with the land, so the neighbor can enforce the covenant as an equitable servitude without resort to implying a reciprocal negative servitude.


Related study sets

Comparing Poetry: Poetic Devices 100%

View Set

BUS498- Strategic Management Exam 1

View Set

Chapter 45: Nursing Care of the Child With an Alteration in Tissue Integrity/Integumentary Disorder

View Set

GENERAL INSURANCE PRINCIPLES class3 INSURANCE PRODUCERS

View Set

aPHR test study guide - Compensation and Benefits (14%)

View Set

Introduction to Entrepreneurship Chapter 5 Quiz MindtapOpportunity identification

View Set

Programming Fundamental quiz 1-4

View Set